(02.02 LC)
If g(x) = x2 + 2, find g(3).
O9
O8
O 11
O6

Answers

Answer 1

Answer:

8

Step-by-step explanation:

g(x) = 2x + 2

g(3) = 2*3 + 2             Substitute 3 for x.

g(3) = 6 + 2                 Multiply.

g(3) = 8                       Add.

Answer 2

Answer:

g(3) = 11

Step-by-step explanation:

Substitute x = 3 into g(x) , that is

g(3) = 3² + 2 = 9 + 2 = 11


Related Questions

9) Jamar drove 228 miles and used 6 gallons of gas.
a) How many miles/gallon did he get on the trip?

b) On another trip, he used 9 gallons of gas. How far did he travel?

Answers

Answer: 38 miles per gallon ; 342 miles.

Step-by-step explanation:

The miles/gallon that he got on the trip will be:

= 228/6

= 38 miles per gallon.

When he used 9 gallons of gas, the distance travelled will be:

= 38 × 9

= 342 miles

Answer:

Question :

Jamar drove 228 miles and used 6 gallons of gas.

a) How many miles/gallon did he get on the trip?b) On another trip, he used 9 gallons of gas. How far did he travel?

Solution :

a) How many miles/gallon did he get on the trip?

[tex]{\implies{\sf{\dfrac{228}{6}}}}[/tex]

[tex]{\implies{\sf{ \cancel{\dfrac{228}{6}}}}}[/tex]

[tex]{\implies{\sf{\underline{\underline{\red{36 \: miles/gallon}}}}}}[/tex]

Hence, he get 38 miles/gallon for his trip.

[tex]\rule{200}2[/tex]

b) On another trip, he used 9 gallons of gas. How far did he travel?

[tex]{\implies{\sf{38 \times 9}}}[/tex]

[tex]{\implies{\sf{\underline{\underline{\red{342 \: miles}}}}}}[/tex]

Hence, he traveled 342 miles by using o gallon og gas.

[tex]\underline{\rule{220pt}{3pt}}[/tex]

Why do we not use a Leg-Leg (LL) Theorem that says if you know that the legs of two right triangles are congruent, then the triangles are congruent?
A. This information is not sufficient to prove triangles congruent.
B. This case is already covered by the HL Theorem.
C. This case is already covered by the SSS Postulate.
D. This case is already covered by the SAS Postulate.

Answers

It’s c I think but correct me if I’m wrong

OMG PLS HURRY IN TEST!!

Select the correct answer.

Which graph represents this system of inequalities?
y > x + 3
y ≤ 5x − 1

Answers

Answer:

A is the correct answer!

Good luck on your test!

Answer: a

Step-by-step explanation:

Y=-3x+1 y+5=-3(x-2) what do the following two equations represent A)The same line B)Distinct parallel lines C)Perpendicular Lines D)Intersecting, but not perpendicular lines

Answers

Answer:

C) perpendicular lines

frEE po ints

Lets say you own a ferret store and you have 20,female ferrets worth $400 and 2, male ferrets worth 350 dollar ferrets. you sell 1male ferret and 8 female ferrets how much money do you get from the sellings?
pt.b
your store gets robbed and you loose 800 dollars worth of female ferrets and 350bucks worth of male ferrets how many ferrets do you have in still have in toatal?

Answers

Answer:

Part A, 3550$

Part B, Ten female ferrets left

Step-by-step explanation:

HELP ME PLEASE..!!!!!! Thank youuuuuu…

Answers

Answer:

22

Step-by-step explanation:

2x+x-2=2x+9

3x-2=2x+9

x-2=9

x=11

MN=2x

MN=2(11)

MN=22

I want help its urgent!!!

(iv), (v), (vi) Questions​

Answers

Solution:

iv) Given equation: [tex]x^{2} - y^{2} - 4x - 2y + 3[/tex]

Since we are given two variables, we need to split the original expression into two quadratic expressions as

[tex]x^{2} - 4x + 4 - y^{2} - 2y - 1[/tex]

Factoring x first:

[tex]x^{2} - 4x + 4 =[/tex] [tex]x^{2} - 2x - 2x + 4[/tex]

= [tex]x (x - 2) - 2 (x-2)[/tex]

= [tex](x-2)(x-2)[/tex]

[tex]= (x-2)^{2}[/tex]

Factoring y now:

[tex]-y^{2} - 2y - 1 = -1(y^{2} + 2y + 1)[/tex]

= [tex]-1(y^{2} + y + y + 1)[/tex]

= [tex]-1[ y(y+1) + 1(y+1)][/tex]

= [tex]-1 [(y+1)(y+1)][/tex]

= [tex](y+1)(-y-1)[/tex]

Therefore, the original expression becomes

[tex](x-2)^{2}+(y+1)(-y-1)\\or \\(x-2)(x-2) + (y+1)(-y-1)[/tex]

The figure below shows a rectangular prism and a cube:

A rectangular prism and a cube are drawn side by side. The length of the rectangle prism is 4 inches, the width is 3 over 4 inch, and its height is 1 and 2 over 4 inches. The cube has a side length of 1 over 4 inch.

How many such cubes are required to completely pack the prism without any gap or overlap?
choices:
288
576
1,152
1,636

Answers

800 cubes is ur answer

Answer:

576

Step-by-step explanation:

find the volume and then divide by the cube

PLEASE NO LINKS AND NO NEGATIVITY

Answers

A=32x-8

A=L*W
L=8x-2
W=4
A=(8x-2)(4)
A=32x-8

5+(3²-4)-6/3 please help

Answers

Answer:8 is the answer.Step-by-step explanation:5 + (3² - 4) - 6/3=> 5 + (9 - 4) - 6/3=> 5 + 5 - 6/3=> 5 + 5 - 2=> 8Conclusion:

Therefore, 8 is the answer.

Hoped this helped.

[tex]BrainiacUser1357[/tex]

Examine the diagram, where BD is tangent to circle M at point D, and BA is secant to the same circle at points E and A.

Answers

Answer:

19 = x

Step-by-step explanation:

The square of the tangent segment = The product of (whole secant segment) (the outside part of the secant segment)

BD²= (BA)(BE)        

8² = (4 + x - 7) (4)             BA = BE + EB

8² = (x - 3)(4)

64 = 4x - 12

64 + 12 = 4x

76 = 4x

19 = x

write the equation of a line that passes through the points (-4,-7) and (-6,9)?

Answers

To solve this you need to use the formula: y2 - y1

                                                                          x2 - x1

9-(-7) over -6-(-4)

The answer would be -8.

Question 14 (5 points)
image

Which of the following is a true statement about the rhombus shown?
Question 14 options:

A)



B)

m∠SPQ = 90°

C)

m∠STP = 90°

D)





Question 15 (5 points)
image

Determine the value of x.
Question 15 options:

A)

x = 90°

B)

x = 45°

C)

x = 20°

D)

x = 4.5°

Answers

Answer:

14) C

15) D, because 90:20=4.5

Pat is saving money to buy a bicycle. The amount he has saved is shown in the table below.





Which of the functions below describe the amount A , in dollars, Pat has saved after t weeks?

Select all that apply.


A(t)=15+15(t−1)

A(t)=30+15(t−1)

A(t)=15+15t

A(t)=15+30t

A(t)=15t

Answers

Answer:

I think A and C

Step-by-step explanation:

PLEASE HELP ME WITH THIS I WILL MARK BRAIINLIST!!

Answers

Answer:

10

Step-by-step explanation:

First solve for x. HI = 2x, IJ = 4x - 10, and the whole length HJ = 4x.

So 2x + 4x - 10 = 4x.

6x - 10 = 4x

6x - 4x - 10 = 4x - 4x

2x - 10 = 0

2x - 10 + 10 = 0 + 10

2x = 10

x = 5.

IJ is 4x - 10, so it becomes 4(5) - 10

20 - 10 = 10

What is the answer I’ll rate you 5

Answers

The equation of the line is [tex]y=8[/tex]

To get the required equation, use the general form of the equation of a line having a gradient [tex]m[/tex], and passing through a point [tex](x_0,y_0)[/tex]

[tex]y-y_0=m(x-x_0)[/tex]

For our problem, the line has [tex]0[/tex] gradient, and passes through the point [tex](4,8)[/tex]

substituting the required values, we have

[tex]y-y_0=m(x-x_0)\\y-8=0(x-4)\\y-8=0\\y=8[/tex]

The required equation is [tex]y=8[/tex]

Learn more here: https://brainly.com/question/1690609

5m - 2 represents the statement two less than five times a number

Answers

Answer:

yes

Step-by-step explanation:

Solve for x help me please****

Answers

Answer:

2x-10=x

take -10 to the other side it becomes +10

take +x to the other side where there is 2x it becomes-xevaluate 2x-x=xso x=10

45 + 30 = 100 - w
Answer correctly luvs

Answers

Answer:

the answer is 25

Step-by-step explanation:

45+30=100-w

then your going to subtract 45 on both sides. So 100-45 is 55.

so now your equation looks like this

30 = 55 - w

subtract 55 on both sides so you will have -25 = -w. Then your going to divide -w on both sides and you will have positive 25

i hope thus helps

Solve the system of equations.
4х – 3y = 12
3х + 6у = 42

Answers

the answer would be B) 6, 4

mark me brianliest if i was correct!

Answer: B. (6,4)

Find x for the 2nd equation

3x + 6y = 42

3x = 42-6y

x= (42-6y) / 3

Place the value of x in the first equation and find for y then.

4x - 3y= 12

4{(42-6y)/3} - 3y = 12

So the answer for y is 4

Now, you know the value of y is 4.

4x - 3*4 = 12

4x - 12 = 12

x= 12+12 /4

x= 6

So x is 6 and y is 4. So the answer is B.

I have to find out which quadratic equations are in standard form?

Answers

Answer:

See image

Step-by-step explanation:

In order to be in standard form, all the variables should be on the same side of the equation. The x^2 term should be first, then the x term then the constant (you could think of it as an x^0 term; it is a plain number) The coefficients of the x^2 term and the x term, should be integer coefficients (the number next to the x^2 and x cannot be a fraction or decimal)

send help pleaseeeeeeeeeeeeee

Answers

MY ANSWER WITH SOLVING STEPS

#CARRYONLEARNING

DON'T FORGET HIT THE THANK YOU BUTTON

She received 900 beads, and 10% of them were green. How many green beads did Liz receive?

Answers

Answer:Liz received about 90 beads that were green.Step-by-step explanation:900 x 10/100 = Total green beads=> 9 x 10 = Total green beads=> 90 = Total green beadsConclusion:

Therefore, Liz received about 90 beads that were green.

Hoped this helped.

[tex]-BrainiacUser1357-[/tex]

Answer:

90

Step-by-step explanation:

10 is 10% of 100 so then 10 times 9 = 90.

please show work

10. If the area of a parallelogram is 690.84 m2 and the height is 20.2 m, what is the length of the base?

Answers

Answer:

34.2 m

Step-by-step explanation:

We know that

Area of parallelogram = Base × Height

690.84 = B × 20.2

690.84/20.2 = B

34.2 = B

Hence,

Base of parallelogram is 34.2 m

[tex]\huge{\purple{\underline{\underline{\bf{\pink{ANSWER:-}}}}}}[/tex]

Here we've been given:

Area of parallelogram (A) = 690.84 m²

Height (H) = 20.2 m

Length of base (B) = ?

[tex]:\longrightarrow\tt{Area \: of \: parallelogram = Base \times Height} \\ \\ :\longrightarrow\tt{690.84 = B \times 20.2} \\ \\ :\longrightarrow\tt{ \frac{690.84}{20.2} = B} \\ \\ :\longrightarrow\tt{34.2 = B} \\ \\ :\longrightarrow\tt{B = 34.2 \: m}[/tex]

The length of base is 34.2m.

Solve each inequality:

a. x + 3 < 4


b. x - 3 > 5

_

c. 4 > x - 2


d. -2 < x + 1

_

Answers

It should be the answers if not sorry
Answer:
A) X<1
B) X>7
C)X<6
D)X>-3

Given −7 is one of the roots of the quadratic equation
(x + k)2 = 16, where k is a constant, find the values of k.

Answers

Answer:

  k = {3, 11}

Step-by-step explanation:

Taking the square root of the given equation, we find ...

  x +k = ±4

For x = -7, this is ...

  -7 +k = ±4

  k = 7 ±4 = {3, 11}

Possible values of k are 3 and 11.

[tex]\huge\bf\underline{\underline{\pink{A}\orange{N}\blue{S}\green{W}\red{E}\purple{R:-}}}[/tex]

Taking square root on the both sides of equation (x + k)² = 16 we get,

[tex]\implies\rm{( {x + k)}^{2} } = 16[/tex]

[tex]\implies\rm{x + k = ± 4}[/tex]

Making two conditions for ± 4 i.e

1) x + k = 4

[tex]\implies\rm{ - 7 + k = 4}[/tex]

[tex]\implies\rm{k = 4 + 7}[/tex]

[tex]\implies\rm{k = 11}[/tex]

2) x + k = -4

[tex]\implies\rm{ - 7 + k = - 4}[/tex]

[tex]\implies\rm{k = - 4 + 7}[/tex]

[tex]\implies\rm{k = 3}[/tex]

Thus the values of k can be either 11 or 3.

State the slope and y-intercept for the graph of
- 8x + y = -12.

Answers

Answer:

y must have a numerical coefficient of 1. This equation does. If it does not, then you must divide through by whatever is in front of the y.

To read the slope, you must have x and its coefficient on opposite sides of the equal sign.

So you get

y = 8x – 12 when you add 8x to both sides

Next, the y intercept is found when x = 0

y = 8x – 12

y = 8(0) – 12

y = – 12

The  y intercept is -12 or in point form (0, – 12)

Answer

slope = 8

y intercept = – 12

Step-by-step explanation:

Find a and b. А E b B 409 с D​

Answers

Answer:

a = 70°, b = 140°

Step-by-step explanation:

use alternate interior angles

The interior angles of a straight line are 180°

a = 180°-110° = 70°

b = 180°-40° = 140°

//I'm sorry, but I'm not very fluent in English.

PLS HELP ASAP ILL GIVE BRAINLKEST PLS THANKS

Answers

Answer: y= (1/4)x

Step-by-step explanation: Best guess given screenshot, matched with desmos.

Do now: Balance the following equation.
1.
C₂H₂ + O₂--
+
--->
- CO2 + H2O

Answers

Answer:

C2H2 + 3O2 ---> 2CO2 + 2H2O

Step-by-step explanation:

Other Questions
Please help me solve this problem ASAP difference between associative and commutative property during which stage of labor is the fetus delivered? how long does it take for dentist numb to wear off I need help Asap!! helpA. character B. plotC settingD. cause and effect What is Plumer's concern about a tie in the Electoral College? Why does he think this would be a bad outcome? Lalit got 50% in Hindi,75% in English and 90 marks in maths. The maximum marks in each subject were 100,140 and 160 respectively. Find his aggregate percentage What is the punishment for a hit and run in the state of california? In what state would someone be least likely to successfully run their home on solar power? California Michigan New Mexico Texas poor ________ can skew election results, particularly if one age or socioeconomic group is more diligent in its efforts to make it to the polls. PLS HELP. NO LINKS. NO SCAM.What is the purpose of the underlined part of the sentence? One of Jin's greatest joys in life is sledding down the hill in the park. A. It is a gerund that names an action.B. It is a participle that modifies the word hill.C. It is a participle that modifies the word down.D. It is a gerund that states who performs an action. which statements accurately reflect the permeability of the cell membrane? choose all that apply Henri nest pas __________ chez son oncle cet t pendant les vacances. The discovery that syphilis led to paralysis and to delusions of grandeur supported the _____ perspective. During a chess match there are 15 pieces left on the board. the ratio of the white pieces to black pieces is 3 : 2 how many white pieces are on the white board? Which of the following is an accurate comparison of the presidents formal and informal powers?Formal powersInformal powersASetting the agenda for CongressGranting pardonsBDeclaring warDelivering the State of the Union addressCVetoing legislationIssuing executive ordersDActing as Commander in ChiefAppointing Supreme Court justices Ebony and Tamika love shoes. When comparing the number of pairs of shoes that they own, they noticed that Ebony has four times as many as Tamika. If Tamika has s pairs of shoes and Ebony has four times as many, who has more? Describe two ways in which family life benefits its members. What is the answer to this question? The motor in a hairdryer is attached to a 3V battery. A current of 0.8A flows through the motor for 3 minutes. Calculate the total charge in this time.